Pain and gain

Algebra Level 5

x 2 y 2 z 2 \large x^2y^2z^2

Let x , y x,y and z z be reals number satisfying the conditions x + y + z = 0 x+y+z=0 and x 2 + y 2 + z 2 = 1 x^2+y^2+z^2=1 .

Let the maximum of the expression above is P P , submit your answer as 10000 P \left \lfloor 10000P \right \rfloor .


The answer is 185.

This section requires Javascript.
You are seeing this because something didn't load right. We suggest you, (a) try refreshing the page, (b) enabling javascript if it is disabled on your browser and, finally, (c) loading the non-javascript version of this page . We're sorry about the hassle.

2 solutions

Otto Bretscher
Feb 20, 2016

We can parameterize this circle as x = a cos ( t ) b sin ( t ) , y = a cos ( t ) b sin ( t ) , z = 2 b sin ( t ) x=a\cos(t)-b\sin(t),y=-a\cos(t)-b\sin(t),z=2b\sin(t) , where a = 1 2 a=\frac{1}{\sqrt{2}} and b = 1 6 b=\frac{1}{\sqrt{6}} . Now x y z = 2 6 ( 1 6 sin 2 ( t ) 1 2 cos 2 ( t ) ) sin ( t ) = 1 3 6 sin ( 3 t ) xyz=\frac{2}{\sqrt{6}}\left(\frac{1}{6}\sin^2(t)-\frac{1}{2}\cos^2(t)\right)\sin(t)=-\frac{1}{3\sqrt{6}}\sin(3t) has a maximal absolute value of 1 3 6 \frac{1}{3\sqrt{6}} , and the maximum of x 2 y 2 z 2 x^2y^2z^2 is 1 54 0.0185 \frac{1}{54}\approx 0.0185 . The answer is 185 \boxed{185}

Alternatively, for a given solution x , y , z x,y,z , consider the polynomial f ( t ) = ( t x ) ( t y ) ( t z ) f(t)=(t-x)(t-y)(t-z) = t 3 1 2 t x y z =t^3-\frac{1}{2}t-xyz , by Viete. Using a little calculus, we observe that the polynomial t 3 1 2 t q t^3-\frac{1}{2}t-q has all real roots if and only if q 1 3 6 |q|\leq\frac{1}{3\sqrt{6}} ; those values of q q are the possible values of x y z xyz .

Oh! Did you, by any chance, get your second solution from my problem? :)

Manuel Kahayon - 5 years, 3 months ago

Log in to reply

That solution was known long before you and I were born ;) It's a standard way to do these kinds of problems, going back at least to Euler

Otto Bretscher - 5 years, 3 months ago

Log in to reply

Ohh... Sorry, not aware of that... I just thought it was something original... It just seemed very similar to my solution...

Manuel Kahayon - 5 years, 3 months ago

Log in to reply

@Manuel Kahayon If you look closely, you will find many problems on Brilliant where that same technique is used... you know, people on Brilliant love Viete ;)

Otto Bretscher - 5 years, 3 months ago

Log in to reply

@Otto Bretscher Aaahh, I didn't mean that part... I meant the part where you use calculus to find the local maxima of the function to minimize the constant term... Or something of the sort...

Manuel Kahayon - 5 years, 3 months ago

Apart from than "using a little calculus", we could also use the cubic discriminant along with the condition for 3 real roots.

Calvin Lin Staff - 5 years, 3 months ago
Pi Han Goh
Apr 24, 2016

Relevant wiki: Cubic Discriminant

Because ( x + y + z ) 2 = x 2 + y 2 + x 2 + 2 ( x y + x z + y z ) (x+y+z)^2 = x^2+y^2 + x^2 + 2(xy+xz+yz) , then x y + x z + y z = 1 2 xy + xz + yz = - \dfrac12 .

Let f ( X ) f(X) be a monic cubic polynomial with roots x , y x,y and z z . Then by Vieta's formula , f ( X ) = X 3 1 2 X + Q f(X) = X^3 - \dfrac12 X + Q , where Q = x y z Q = -xyz . So we want to maximize Q 2 = P Q^2 = P .

Since all it's roots are real numbers, then its cubic discriminant is non-negative:

b 2 c 2 4 a c 3 4 b 3 d 27 a 2 d 2 + 18 a b c d 0 , b^2 c^2 -4ac^3 - 4b^3 d - 27a^2 d^2 + 18abcd \geq 0 \; ,

with a = 0 , b = 0 , c = 1 2 , d = Q a= 0, b = 0, c = -\dfrac12, d = Q . Substituting these values and simplifying it gives Q 2 1 54 Q^2 \leq \dfrac1{54} . So P = 1 54 P = \dfrac1{54} , and so our answer is 10000 P = 10000 54 = 185 \lfloor 10000 P \rfloor = \left \lfloor \dfrac{10000}{54} \right \rfloor = \boxed{185} .

0 pending reports

×

Problem Loading...

Note Loading...

Set Loading...